Howdy, Stranger!

It looks like you're new here. If you want to get involved, click one of these buttons!

Question stem misleads about type of question? [Throws up hands, gives up]

steve-10steve-10 Alum Member
Near the start of this video lesson in the Causation and Phenomenon-Hypothesis Questions group, JY points out that a stem which says, "Which of the following, if true, most strongly supports the ... argument?" is actually a strengthening question and admonishes us not to confuse it with a MSS stem!

Yes, he gives a persuasive reason -- the direction of support is here consistent with strengthening and not with MSS -- but what the hell? If the stem isn't going to indicate the type of the question, why are we reading it first and using it to guide reading of the stimulus?

Comments

  • Tinyosi1Tinyosi1 Alum Member
    235 karma
    You still need to read the question stem to know exactly what you are meant to do; and the question stem indicates perfectly what type of question it is. The point isn't that the question stem tricks you or doesn't reveal the question type. The point is that the phrasing of the question stem, while similar to MSS, is indeed astrengthening question due to the way the sentence reads.

    In short, JY's point isn't that the stem completely hides the nature of the question; his point is that some people might be tripped up by the wording. A person with fully developed reading comprehension won't be confused by the sentence; his warning would apply more to people who might read too fast or who still have a little bit to go in that area.

    I wouldn't give this a huge amount of thought; if the sentence trips you up now just continue to develop your reading comprehension skill as well as your familiarity with the LSAT question types and soon enough these kind of questions will be no issue! I hope I covered all of the bases and I hope you understand what I mean.
  • steve-10steve-10 Alum Member
    192 karma
    I'm still wondering about the necessity (?) of reading the question stem first. If we have to read the stimulus to understand the stem, why not just start with the stimulus?
  • steve-10steve-10 Alum Member
    192 karma
    (It'd be nice if there were editing allowable here as there is for lesson comments.)

    OK, I think I see that the stem indicates what's going on by its reference to what is going to provide the support, the stimulus or the AC.
  • inactiveinactive Alum Member
    12637 karma
    @"steve-10" said:
    (It'd be nice if there were editing allowable here as there is for lesson comments.)
    Psst, editing IS allowed. ;)
    http://content.screencast.com/users/spaceursa/folders/Snagit/media/76d4f531-c731-405a-8b39-4308268ec9f2/11.25.2016-17.37.png
  • SamiSami Live Member Sage 7Sage Tutor
    edited November 2016 10774 karma
    @"steve-10" said:
    Which of the following, if true, most strongly supports the ... argument?

    Hi Steve, so just by reading the stem I knew it was a strengthening question.
    Here is the difference:
    1) Which one of the following, if true, most strongly supports the argument?
    2) Which one of the following is most strongly supported by the argument?

    In the first instance, the question is asking us, which one of the following arguments below, taken as true, will support the argument above? (so the answer choice will help the stimulus/argument above) This is an example of strengthening question stem.
    In the second instance you are being asked which one of the following statements below is supported by the argument/stimulus above? This is an example of most strongly supported question stem.

    So although they look similar when read they actually mean different things.
    @"steve-10" said:
    I'm still wondering about the necessity (?) of reading the question stem first. If we have to read the stimulus to understand the stem, why not just start with the stimulus?
    I can't remember an instance where I had to read the stimulus to understand the type of operation LSAT expected me to perform on it. So reading it before hand is extremely helpful as I keep in mind what my task is in the stimulus.

  • AlexAlex Alum Member
    23929 karma
    @Sami said:
    Which of the following, if true, most strongly supports the ... argument?
    Hi Steve, so just by reading the stem I knew it was a strengthening question.
    Here is the difference:
    1) Which one of the following, if true, most strongly supports the argument?
    2) Which one of the following is most strongly supported by the argument?




    Precisely!
    @Tinyosi1 said:
    I wouldn't give this a huge amount of thought; if the sentence trips you up now just continue to develop your reading comprehension skill as well as your familiarity with the LSAT question types and soon enough these kind of questions will be no issue! I hope I covered all of the bases and I hope you understand what I mean.
    Tinyosi1 nailed it too. That same phrasing used to mess with me when I first began. After a while you'll look back and laugh that this ever confused ya. Just takes some time :)
  • steve-10steve-10 Alum Member
    192 karma
    @"steve-10" said:
    OK, I think I see that the stem indicates what's going on by its reference to what is going to provide the support, the stimulus or the AC.
  • Cant Get RightCant Get Right Yearly + Live Member Sage 🍌 7Sage Tutor
    27809 karma
    They're not above confusing/misleading question stems though. I found PT 78 LR to be really tricky on this. In review, there were several questions where I realized that I'd functionally misidentified question stems.
Sign In or Register to comment.